서브 프로 럼과 하위 주문 연기 조건 사이의 비 일정한 중량 재표를 가진 분열 및 정복 유형 재발의 점근증

cs.stackexchange https://cs.stackexchange.com/questions/126572

  •  29-09-2020
  •  | 
  •  

문제

알고리즘의 런타임을 분석하려고하면 다음 유형의 재발에 도착합니다.

$$ \ begin {사례} t (n)=theta (1), & \ text {작은 $ n $;} \\ T (n) \ LEQ T (A_N n + h (n)) + T (((1-a_n) n + h (n) + f (n), & \ text {큰 $ n $의 경우.} \ end {사례} $$ 여기서,

  • $ a_n $ 은 알 수 없으며 $ n $ 에 의존 할 수 있지만 두 상수 < SPAN 클래스="수학 용기"> $ 0
  • $ h (n) $ h (n) $ h (n) $ n $ ( $ o (n ^ \ epsilon) $ $ 0 \ leq \ epsilon <1 $ ) .

$ a_n $ 이 상수 였고 Akra-bazzi 메소드를 사용하여 결과를 얻을 수 있습니다. 한편, 퍼지 용어가 존재하지 않으면 재귀 트리 분석의 일부 유형은 직선적입니다.

조금 더 많은 콘크리트를 만들려면 여기서는 점근증을 얻고 싶은 재발입니다.

$$ \ begin {사례} T (n)= 1, & \ text {n= 1;} \\ T (n) \ LEQ T (A_N n + \ sqrt n) + t ((1-a_n) n + \ sqrt n) + n, & \ text {$ n \ geq 2 $} \ end {사례} $$ 여기서 $ \ fRAC {1} {4} \ LEQ A_N \ LEQ \ FRAC {3} {SPAN CLASS="수학 컨테이너에 대한 모든 <4} $ >> $ n \ geq 1 $

나는 상한이나 변형에 대한 다양한 추측을 시도했다. 모든 것이 $ o (n \ log (n)) $ 에 이르기까지 $ o (n ^ {1+ \ epsilon}) $ ( $ \ epsilon> 0) 만 증명할 수 있습니다. $ ), 이것은 마스터 이론 à 라 아크라 - Bazzi의 일반화가 보살핌을받을 수 있어야하는 것과 같습니다.

이러한 유형의 재발을 해결하는 방법에 대한 제안은 무엇입니까?

도움이 되었습니까?

해결책

op에 따르면, 증거를 완성하기 위해 우리는 충분히 큰 $ n $ 을 증명해야합니다. $$ (A_N N + \ SQRT {n}) ^ {A_N + 1 / \ sqrt {n}} ((1-a_n) n + sqrt {n}) ^ {1-a_n + 1 / \ sqrt {n}} \ leq n. $$ $ n ^ {1 + 2 / \ sqrt {n}} $ , 우리는 얻는다. $$ (a_n + 1 / \ sqrt {n}) ^ {a_n + 1 / \ sqrt {n}} (1-a_n + 1 / \ sqrt {n}) ^ {1-a_n + 1 / \ sqrt {n}} \ leq n ^ {- 2 / \ sqrt {n}}. $$ $ (1 + 2 / sqrt {n}) ^ {1 + 2 / \ sqrt {n}} $ , 우리는 얻는다 $$ \ left (\ frac {a_n + 1 / \ sqrt {n}} {1 + 2 / \ sqrt {n}} \ right) ^ {a_n + 1 / \ sqrt {n}} \ left (\ frac {1-a_n + 1 / \ sqrt {n}} {1 + 2 / \ sqrt {n}} \ sqrt ^ {1-a_n + 1 / \ sqrt {n}} \ LEQ \ frac {1} {n ^ {2 / \ sqrt {n}} (1 + 2 / \ sqrt {n}) ^ {1 + 2 / \ sqrt {n}}}}. $$ $ 1 / (1 + 2 / \ sqrt {n}) $ $ p= (a_n + 1 / \ sqrt {n} / (1 + 2 / \ sqrt {n}) $ , 우리는 얻는다 $$ p ^ p (1-p) ^ {1-p} \ Leq \ frac {1} {n {(2 / \ sqrt {n}) / (1 + 2 / \ sqrt {n}) (1+ 2 / \ sqrt {n})}. $$

왼쪽면은 $ 1 / \ exp h (p) $ , 여기서 $ h (p) $ 은 엔트로피 기능입니다. 따라서 $ a_n= 1/4 $ 을 할 때 최대화됩니다. $ p \ 약 1/4 $ $ (1/4) ^ {1/4} (3/4) ^ {3/4} <1 $ , 충분히 큰 $ n $ 우리는 일부 $ \ theta <1 $ 에 의해 왼쪽을 묶을 수 있습니다. 오른쪽에 관해서는 $ n \ \ infty $ $ 1 $ 경향이 있습니다. 실제로, 대수를 찍고, 우리는 얻습니다 $$ \ FRAC {2 \ log n} {\ sqrt {n} + 2} + o \ left (\ frac1 {\ sqrt {n}} \ 오른쪽) \ Liggrightarrow 0. $$ 특히 $ n $ $ \ theta $ 보다 큰 것입니다.

라이센스 : CC-BY-SA ~와 함께 속성
제휴하지 않습니다 cs.stackexchange
scroll top